Find the area enclosed by circles $r=3costheta$ and $r=3sintheta$

The name of the pictureThe name of the pictureThe name of the pictureClash Royale CLAN TAG#URR8PPP











up vote
0
down vote

favorite
1












Find the area enclosed by circles $r=3costheta$ and $r=3sintheta$



2-circles



I know that the graph of $r=3sintheta$ intersects at $theta=0$ and at $theta=pi$ why aren't these two bounds used and why only $theta$ equaling $pi$? Also the graph $r=3costheta$ intersects the pole at $theta=frac3pi2,frac1pi2$. So why do they only use $fracpi2$?







share|cite|improve this question























    up vote
    0
    down vote

    favorite
    1












    Find the area enclosed by circles $r=3costheta$ and $r=3sintheta$



    2-circles



    I know that the graph of $r=3sintheta$ intersects at $theta=0$ and at $theta=pi$ why aren't these two bounds used and why only $theta$ equaling $pi$? Also the graph $r=3costheta$ intersects the pole at $theta=frac3pi2,frac1pi2$. So why do they only use $fracpi2$?







    share|cite|improve this question





















      up vote
      0
      down vote

      favorite
      1









      up vote
      0
      down vote

      favorite
      1






      1





      Find the area enclosed by circles $r=3costheta$ and $r=3sintheta$



      2-circles



      I know that the graph of $r=3sintheta$ intersects at $theta=0$ and at $theta=pi$ why aren't these two bounds used and why only $theta$ equaling $pi$? Also the graph $r=3costheta$ intersects the pole at $theta=frac3pi2,frac1pi2$. So why do they only use $fracpi2$?







      share|cite|improve this question











      Find the area enclosed by circles $r=3costheta$ and $r=3sintheta$



      2-circles



      I know that the graph of $r=3sintheta$ intersects at $theta=0$ and at $theta=pi$ why aren't these two bounds used and why only $theta$ equaling $pi$? Also the graph $r=3costheta$ intersects the pole at $theta=frac3pi2,frac1pi2$. So why do they only use $fracpi2$?









      share|cite|improve this question










      share|cite|improve this question




      share|cite|improve this question









      asked Jul 24 at 0:10









      Jinzu

      328311




      328311




















          3 Answers
          3






          active

          oldest

          votes

















          up vote
          2
          down vote



          accepted










          The two circles are intersecting at $theta=dfracpi4$ and $r=0$



          The area region can be divided into two parts.
          enter image description here



          Note that we are bounded by the region of the red circle $($the lower part of the half$)$ by $theta=0mbox to theta=dfracpi4$



          Similarly, the region of the purple circle $($the upper part of the half$)$ is bounded by $theta=dfracpi4$ to $dfracpi2$



          So, we get $$dfrac12int_0^dfracpi4(3sintheta)^2dtheta+dfrac12int_fracpi4^dfracpi2(3costheta)^2dtheta$$






          share|cite|improve this answer























          • Perhaps you should proofread? It's also worth commenting that the two circles intersect at $r=0$, even though the $theta$-values are different.
            – Ted Shifrin
            Jul 24 at 0:28











          • Great, but you still have a bunch of stuff to fix.
            – Ted Shifrin
            Jul 24 at 0:34










          • Did you look at the sum of integrals you wrote down? It's a matter of having something correct, not a matter of adding.
            – Ted Shifrin
            Jul 24 at 0:36











          • Agh. You need to be more self-critical. The integrand in the second integral is still wrong.
            – Ted Shifrin
            Jul 24 at 0:56










          • @TedShifrin Fixed
            – Key Flex
            Jul 24 at 0:57

















          up vote
          0
          down vote













          Answer to the question: "Why do they only use $pi/2$... ?"



          Note that the intersection region is locate in the first quadrant. This is parametrized by $theta in left[0,;frac pi2right]$.



          Consider now a "radar ray" of an angle $theta$ in this interval. Then it first determines an $r$-interval in the intersection domain, and we know its limits. So the integral is
          $$
          beginaligned
          J &=int_theta in [0,;pi/2]
          int_rin[ 0,; 3min(costheta,; sintheta) ]r; dr; dtheta
          \
          &=
          int_theta in [0,;pi/2]
          left[frac r^22 right]_r=0^r=3min(costheta,;sintheta); dtheta
          \
          &=
          2int_theta in [0,;pi/4]
          frac 12cdot 9sin^2theta; dtheta
          \
          &qquadtext(use the substitution $y=pi/2-x$ for the
          interval $left[fracpi4,;fracpi2right]$)
          \
          &=
          frac 98(pi-2)
          endaligned
          $$
          The result fits with the geometrical intuition, we compute the double of the following:



          • the area of a quarter of a disk with radius $frac 32$,


          • and from it we remove half of the area of a square with side $frac 32$. I.e.
            $$
            2left( frac 14pileft(frac 32right)^2-frac 12left(frac 32right)^2 right) .
            $$






          share|cite|improve this answer




























            up vote
            0
            down vote













            No calculus:



            Let the two intersection points of the circles be A and B do that A is (0, 0) and B is (1.5, 1.5). It's easy to see that the arc formed by the center of a circle and A and B has measure $pi/2$. So the area of that sector is $2.25pi/4$. The area of the triangle formed by A, B, and center of the circle is $2.25/2$. So the area of half the desired area is $2.25pi/4 -2.25/2 $ and the desired area is $5pi/4-2.25$






            share|cite|improve this answer





















              Your Answer




              StackExchange.ifUsing("editor", function ()
              return StackExchange.using("mathjaxEditing", function ()
              StackExchange.MarkdownEditor.creationCallbacks.add(function (editor, postfix)
              StackExchange.mathjaxEditing.prepareWmdForMathJax(editor, postfix, [["$", "$"], ["\\(","\\)"]]);
              );
              );
              , "mathjax-editing");

              StackExchange.ready(function()
              var channelOptions =
              tags: "".split(" "),
              id: "69"
              ;
              initTagRenderer("".split(" "), "".split(" "), channelOptions);

              StackExchange.using("externalEditor", function()
              // Have to fire editor after snippets, if snippets enabled
              if (StackExchange.settings.snippets.snippetsEnabled)
              StackExchange.using("snippets", function()
              createEditor();
              );

              else
              createEditor();

              );

              function createEditor()
              StackExchange.prepareEditor(
              heartbeatType: 'answer',
              convertImagesToLinks: true,
              noModals: false,
              showLowRepImageUploadWarning: true,
              reputationToPostImages: 10,
              bindNavPrevention: true,
              postfix: "",
              noCode: true, onDemand: true,
              discardSelector: ".discard-answer"
              ,immediatelyShowMarkdownHelp:true
              );



              );








               

              draft saved


              draft discarded


















              StackExchange.ready(
              function ()
              StackExchange.openid.initPostLogin('.new-post-login', 'https%3a%2f%2fmath.stackexchange.com%2fquestions%2f2860889%2ffind-the-area-enclosed-by-circles-r-3-cos-theta-and-r-3-sin-theta%23new-answer', 'question_page');

              );

              Post as a guest






























              3 Answers
              3






              active

              oldest

              votes








              3 Answers
              3






              active

              oldest

              votes









              active

              oldest

              votes






              active

              oldest

              votes








              up vote
              2
              down vote



              accepted










              The two circles are intersecting at $theta=dfracpi4$ and $r=0$



              The area region can be divided into two parts.
              enter image description here



              Note that we are bounded by the region of the red circle $($the lower part of the half$)$ by $theta=0mbox to theta=dfracpi4$



              Similarly, the region of the purple circle $($the upper part of the half$)$ is bounded by $theta=dfracpi4$ to $dfracpi2$



              So, we get $$dfrac12int_0^dfracpi4(3sintheta)^2dtheta+dfrac12int_fracpi4^dfracpi2(3costheta)^2dtheta$$






              share|cite|improve this answer























              • Perhaps you should proofread? It's also worth commenting that the two circles intersect at $r=0$, even though the $theta$-values are different.
                – Ted Shifrin
                Jul 24 at 0:28











              • Great, but you still have a bunch of stuff to fix.
                – Ted Shifrin
                Jul 24 at 0:34










              • Did you look at the sum of integrals you wrote down? It's a matter of having something correct, not a matter of adding.
                – Ted Shifrin
                Jul 24 at 0:36











              • Agh. You need to be more self-critical. The integrand in the second integral is still wrong.
                – Ted Shifrin
                Jul 24 at 0:56










              • @TedShifrin Fixed
                – Key Flex
                Jul 24 at 0:57














              up vote
              2
              down vote



              accepted










              The two circles are intersecting at $theta=dfracpi4$ and $r=0$



              The area region can be divided into two parts.
              enter image description here



              Note that we are bounded by the region of the red circle $($the lower part of the half$)$ by $theta=0mbox to theta=dfracpi4$



              Similarly, the region of the purple circle $($the upper part of the half$)$ is bounded by $theta=dfracpi4$ to $dfracpi2$



              So, we get $$dfrac12int_0^dfracpi4(3sintheta)^2dtheta+dfrac12int_fracpi4^dfracpi2(3costheta)^2dtheta$$






              share|cite|improve this answer























              • Perhaps you should proofread? It's also worth commenting that the two circles intersect at $r=0$, even though the $theta$-values are different.
                – Ted Shifrin
                Jul 24 at 0:28











              • Great, but you still have a bunch of stuff to fix.
                – Ted Shifrin
                Jul 24 at 0:34










              • Did you look at the sum of integrals you wrote down? It's a matter of having something correct, not a matter of adding.
                – Ted Shifrin
                Jul 24 at 0:36











              • Agh. You need to be more self-critical. The integrand in the second integral is still wrong.
                – Ted Shifrin
                Jul 24 at 0:56










              • @TedShifrin Fixed
                – Key Flex
                Jul 24 at 0:57












              up vote
              2
              down vote



              accepted







              up vote
              2
              down vote



              accepted






              The two circles are intersecting at $theta=dfracpi4$ and $r=0$



              The area region can be divided into two parts.
              enter image description here



              Note that we are bounded by the region of the red circle $($the lower part of the half$)$ by $theta=0mbox to theta=dfracpi4$



              Similarly, the region of the purple circle $($the upper part of the half$)$ is bounded by $theta=dfracpi4$ to $dfracpi2$



              So, we get $$dfrac12int_0^dfracpi4(3sintheta)^2dtheta+dfrac12int_fracpi4^dfracpi2(3costheta)^2dtheta$$






              share|cite|improve this answer















              The two circles are intersecting at $theta=dfracpi4$ and $r=0$



              The area region can be divided into two parts.
              enter image description here



              Note that we are bounded by the region of the red circle $($the lower part of the half$)$ by $theta=0mbox to theta=dfracpi4$



              Similarly, the region of the purple circle $($the upper part of the half$)$ is bounded by $theta=dfracpi4$ to $dfracpi2$



              So, we get $$dfrac12int_0^dfracpi4(3sintheta)^2dtheta+dfrac12int_fracpi4^dfracpi2(3costheta)^2dtheta$$







              share|cite|improve this answer















              share|cite|improve this answer



              share|cite|improve this answer








              edited Jul 24 at 0:57


























              answered Jul 24 at 0:20









              Key Flex

              4,233423




              4,233423











              • Perhaps you should proofread? It's also worth commenting that the two circles intersect at $r=0$, even though the $theta$-values are different.
                – Ted Shifrin
                Jul 24 at 0:28











              • Great, but you still have a bunch of stuff to fix.
                – Ted Shifrin
                Jul 24 at 0:34










              • Did you look at the sum of integrals you wrote down? It's a matter of having something correct, not a matter of adding.
                – Ted Shifrin
                Jul 24 at 0:36











              • Agh. You need to be more self-critical. The integrand in the second integral is still wrong.
                – Ted Shifrin
                Jul 24 at 0:56










              • @TedShifrin Fixed
                – Key Flex
                Jul 24 at 0:57
















              • Perhaps you should proofread? It's also worth commenting that the two circles intersect at $r=0$, even though the $theta$-values are different.
                – Ted Shifrin
                Jul 24 at 0:28











              • Great, but you still have a bunch of stuff to fix.
                – Ted Shifrin
                Jul 24 at 0:34










              • Did you look at the sum of integrals you wrote down? It's a matter of having something correct, not a matter of adding.
                – Ted Shifrin
                Jul 24 at 0:36











              • Agh. You need to be more self-critical. The integrand in the second integral is still wrong.
                – Ted Shifrin
                Jul 24 at 0:56










              • @TedShifrin Fixed
                – Key Flex
                Jul 24 at 0:57















              Perhaps you should proofread? It's also worth commenting that the two circles intersect at $r=0$, even though the $theta$-values are different.
              – Ted Shifrin
              Jul 24 at 0:28





              Perhaps you should proofread? It's also worth commenting that the two circles intersect at $r=0$, even though the $theta$-values are different.
              – Ted Shifrin
              Jul 24 at 0:28













              Great, but you still have a bunch of stuff to fix.
              – Ted Shifrin
              Jul 24 at 0:34




              Great, but you still have a bunch of stuff to fix.
              – Ted Shifrin
              Jul 24 at 0:34












              Did you look at the sum of integrals you wrote down? It's a matter of having something correct, not a matter of adding.
              – Ted Shifrin
              Jul 24 at 0:36





              Did you look at the sum of integrals you wrote down? It's a matter of having something correct, not a matter of adding.
              – Ted Shifrin
              Jul 24 at 0:36













              Agh. You need to be more self-critical. The integrand in the second integral is still wrong.
              – Ted Shifrin
              Jul 24 at 0:56




              Agh. You need to be more self-critical. The integrand in the second integral is still wrong.
              – Ted Shifrin
              Jul 24 at 0:56












              @TedShifrin Fixed
              – Key Flex
              Jul 24 at 0:57




              @TedShifrin Fixed
              – Key Flex
              Jul 24 at 0:57










              up vote
              0
              down vote













              Answer to the question: "Why do they only use $pi/2$... ?"



              Note that the intersection region is locate in the first quadrant. This is parametrized by $theta in left[0,;frac pi2right]$.



              Consider now a "radar ray" of an angle $theta$ in this interval. Then it first determines an $r$-interval in the intersection domain, and we know its limits. So the integral is
              $$
              beginaligned
              J &=int_theta in [0,;pi/2]
              int_rin[ 0,; 3min(costheta,; sintheta) ]r; dr; dtheta
              \
              &=
              int_theta in [0,;pi/2]
              left[frac r^22 right]_r=0^r=3min(costheta,;sintheta); dtheta
              \
              &=
              2int_theta in [0,;pi/4]
              frac 12cdot 9sin^2theta; dtheta
              \
              &qquadtext(use the substitution $y=pi/2-x$ for the
              interval $left[fracpi4,;fracpi2right]$)
              \
              &=
              frac 98(pi-2)
              endaligned
              $$
              The result fits with the geometrical intuition, we compute the double of the following:



              • the area of a quarter of a disk with radius $frac 32$,


              • and from it we remove half of the area of a square with side $frac 32$. I.e.
                $$
                2left( frac 14pileft(frac 32right)^2-frac 12left(frac 32right)^2 right) .
                $$






              share|cite|improve this answer

























                up vote
                0
                down vote













                Answer to the question: "Why do they only use $pi/2$... ?"



                Note that the intersection region is locate in the first quadrant. This is parametrized by $theta in left[0,;frac pi2right]$.



                Consider now a "radar ray" of an angle $theta$ in this interval. Then it first determines an $r$-interval in the intersection domain, and we know its limits. So the integral is
                $$
                beginaligned
                J &=int_theta in [0,;pi/2]
                int_rin[ 0,; 3min(costheta,; sintheta) ]r; dr; dtheta
                \
                &=
                int_theta in [0,;pi/2]
                left[frac r^22 right]_r=0^r=3min(costheta,;sintheta); dtheta
                \
                &=
                2int_theta in [0,;pi/4]
                frac 12cdot 9sin^2theta; dtheta
                \
                &qquadtext(use the substitution $y=pi/2-x$ for the
                interval $left[fracpi4,;fracpi2right]$)
                \
                &=
                frac 98(pi-2)
                endaligned
                $$
                The result fits with the geometrical intuition, we compute the double of the following:



                • the area of a quarter of a disk with radius $frac 32$,


                • and from it we remove half of the area of a square with side $frac 32$. I.e.
                  $$
                  2left( frac 14pileft(frac 32right)^2-frac 12left(frac 32right)^2 right) .
                  $$






                share|cite|improve this answer























                  up vote
                  0
                  down vote










                  up vote
                  0
                  down vote









                  Answer to the question: "Why do they only use $pi/2$... ?"



                  Note that the intersection region is locate in the first quadrant. This is parametrized by $theta in left[0,;frac pi2right]$.



                  Consider now a "radar ray" of an angle $theta$ in this interval. Then it first determines an $r$-interval in the intersection domain, and we know its limits. So the integral is
                  $$
                  beginaligned
                  J &=int_theta in [0,;pi/2]
                  int_rin[ 0,; 3min(costheta,; sintheta) ]r; dr; dtheta
                  \
                  &=
                  int_theta in [0,;pi/2]
                  left[frac r^22 right]_r=0^r=3min(costheta,;sintheta); dtheta
                  \
                  &=
                  2int_theta in [0,;pi/4]
                  frac 12cdot 9sin^2theta; dtheta
                  \
                  &qquadtext(use the substitution $y=pi/2-x$ for the
                  interval $left[fracpi4,;fracpi2right]$)
                  \
                  &=
                  frac 98(pi-2)
                  endaligned
                  $$
                  The result fits with the geometrical intuition, we compute the double of the following:



                  • the area of a quarter of a disk with radius $frac 32$,


                  • and from it we remove half of the area of a square with side $frac 32$. I.e.
                    $$
                    2left( frac 14pileft(frac 32right)^2-frac 12left(frac 32right)^2 right) .
                    $$






                  share|cite|improve this answer













                  Answer to the question: "Why do they only use $pi/2$... ?"



                  Note that the intersection region is locate in the first quadrant. This is parametrized by $theta in left[0,;frac pi2right]$.



                  Consider now a "radar ray" of an angle $theta$ in this interval. Then it first determines an $r$-interval in the intersection domain, and we know its limits. So the integral is
                  $$
                  beginaligned
                  J &=int_theta in [0,;pi/2]
                  int_rin[ 0,; 3min(costheta,; sintheta) ]r; dr; dtheta
                  \
                  &=
                  int_theta in [0,;pi/2]
                  left[frac r^22 right]_r=0^r=3min(costheta,;sintheta); dtheta
                  \
                  &=
                  2int_theta in [0,;pi/4]
                  frac 12cdot 9sin^2theta; dtheta
                  \
                  &qquadtext(use the substitution $y=pi/2-x$ for the
                  interval $left[fracpi4,;fracpi2right]$)
                  \
                  &=
                  frac 98(pi-2)
                  endaligned
                  $$
                  The result fits with the geometrical intuition, we compute the double of the following:



                  • the area of a quarter of a disk with radius $frac 32$,


                  • and from it we remove half of the area of a square with side $frac 32$. I.e.
                    $$
                    2left( frac 14pileft(frac 32right)^2-frac 12left(frac 32right)^2 right) .
                    $$







                  share|cite|improve this answer













                  share|cite|improve this answer



                  share|cite|improve this answer











                  answered Jul 24 at 1:53









                  dan_fulea

                  4,1271211




                  4,1271211




















                      up vote
                      0
                      down vote













                      No calculus:



                      Let the two intersection points of the circles be A and B do that A is (0, 0) and B is (1.5, 1.5). It's easy to see that the arc formed by the center of a circle and A and B has measure $pi/2$. So the area of that sector is $2.25pi/4$. The area of the triangle formed by A, B, and center of the circle is $2.25/2$. So the area of half the desired area is $2.25pi/4 -2.25/2 $ and the desired area is $5pi/4-2.25$






                      share|cite|improve this answer

























                        up vote
                        0
                        down vote













                        No calculus:



                        Let the two intersection points of the circles be A and B do that A is (0, 0) and B is (1.5, 1.5). It's easy to see that the arc formed by the center of a circle and A and B has measure $pi/2$. So the area of that sector is $2.25pi/4$. The area of the triangle formed by A, B, and center of the circle is $2.25/2$. So the area of half the desired area is $2.25pi/4 -2.25/2 $ and the desired area is $5pi/4-2.25$






                        share|cite|improve this answer























                          up vote
                          0
                          down vote










                          up vote
                          0
                          down vote









                          No calculus:



                          Let the two intersection points of the circles be A and B do that A is (0, 0) and B is (1.5, 1.5). It's easy to see that the arc formed by the center of a circle and A and B has measure $pi/2$. So the area of that sector is $2.25pi/4$. The area of the triangle formed by A, B, and center of the circle is $2.25/2$. So the area of half the desired area is $2.25pi/4 -2.25/2 $ and the desired area is $5pi/4-2.25$






                          share|cite|improve this answer













                          No calculus:



                          Let the two intersection points of the circles be A and B do that A is (0, 0) and B is (1.5, 1.5). It's easy to see that the arc formed by the center of a circle and A and B has measure $pi/2$. So the area of that sector is $2.25pi/4$. The area of the triangle formed by A, B, and center of the circle is $2.25/2$. So the area of half the desired area is $2.25pi/4 -2.25/2 $ and the desired area is $5pi/4-2.25$







                          share|cite|improve this answer













                          share|cite|improve this answer



                          share|cite|improve this answer











                          answered Jul 24 at 2:28









                          Shrey Joshi

                          1389




                          1389






















                               

                              draft saved


                              draft discarded


























                               


                              draft saved


                              draft discarded














                              StackExchange.ready(
                              function ()
                              StackExchange.openid.initPostLogin('.new-post-login', 'https%3a%2f%2fmath.stackexchange.com%2fquestions%2f2860889%2ffind-the-area-enclosed-by-circles-r-3-cos-theta-and-r-3-sin-theta%23new-answer', 'question_page');

                              );

                              Post as a guest













































































                              Comments

                              Popular posts from this blog

                              Color the edges and diagonals of a regular polygon

                              Relationship between determinant of matrix and determinant of adjoint?

                              What is the equation of a 3D cone with generalised tilt?